Add solutions and graphics to Serway and Jewett v8's chapter 26 problems.
[course.git] / latex / problems / Serway_and_Jewett_8 / problem26.16.tex
1 \begin{problem*}{26.16}
2 Given a $2.50\U{$\mu$F}$ capacitor, a $6.25\U{$\mu$F}$ capacitor, and
3 a $6.00\U{V}$ battery, find the charge on each capacitor if you
4 connect them \Part{a} in series across the battery and \Part{b} in
5 parallel across the battery.
6 \end{problem*}
7
8 \begin{solution}
9 \begin{center}
10 \begin{asy}
11 import Circ;
12
13 real u = 1cm;
14
15 TwoTerminal C1 = capacitor((0,0), ang=0, "$C_1$", "2.50\U{$\mu$F}");
16 TwoTerminal C2 = capacitor(C1.end + (u,0), ang=0, "$C_2$", "6.25\U{$\mu$F}");
17 TwoTerminal B = battery("$V$", "6.00\U{V}", draw=false);
18 B.centerto(C1.beg, C2.end, offset=2u);
19 B.draw();
20 wire(B.beg, C1.beg, rlsq);
21 wire(B.end, C2.end, rlsq);
22 wire(C1.end, C2.beg);
23
24 label("\Part{a}", B.mid + (0, -5u));
25
26 B.shift((4u, 0));
27 B.draw();
28 centerto(B, C1, offset=-2u);
29 C1.draw();
30 centerto(B, C2, offset=-4u);
31 C2.draw();
32 wire(B.beg, C1.beg, rlsq, dist=-u/2);
33 wire(B.beg, C2.beg, rlsq, dist=-u/2);
34 wire(B.end, C1.end, rlsq, dist=u/2);
35 wire(B.end, C2.end, rlsq, dist=u/2);
36
37 label("\Part{b}", B.mid + (0, -5u));
38 \end{asy}
39 \end{center}
40
41 \Part{a}
42 The net capacitance of the two capacitors in series is
43 \begin{equation}
44   C = \p({\frac{1}{C_1} + \frac{1}{C_2}})^{-1}
45     = 1.79\U{$\mu$F} \;.
46 \end{equation}
47 The equivalent capacitor has the total battery voltage across itself,
48 so it carries a charge of
49 \begin{equation}
50   Q = CV = 1.79\U{$\mu$F} \cdot 6.00\U{V} = \ans{10.7\U{$\mu$C}} \;.
51 \end{equation}
52 Because $C_1$ and $C_2$ are in series, they each have the same $Q$ as
53 the equivalent capacitor.
54
55 \Part{b}
56 When the capacitors are in parallel, they each have the total battery
57 voltage, so
58 \begin{align}
59   Q_1 &= C_1 V = 2.50\U{$\mu$F} \cdot 6.00\U{V} = \ans{15.0\U{$\mu$C}} \\
60   Q_2 &= C_2 V = 6.25\U{$\mu$F} \cdot 6.00\U{V} = \ans{37.5\U{$\mu$C}} \;.
61 \end{align}
62 \end{solution}